Difference between revisions of "2017 AMC 10A Problems/Problem 24"

(Solution)
m (Video Solution)
(34 intermediate revisions by 23 users not shown)
Line 4: Line 4:
 
<math>\textbf{(A)}\ -9009 \qquad\textbf{(B)}\ -8008 \qquad\textbf{(C)}\ -7007 \qquad\textbf{(D)}\ -6006 \qquad\textbf{(E)}\ -5005</math>
 
<math>\textbf{(A)}\ -9009 \qquad\textbf{(B)}\ -8008 \qquad\textbf{(C)}\ -7007 \qquad\textbf{(D)}\ -6006 \qquad\textbf{(E)}\ -5005</math>
  
==Solution==
+
==Solution 1==
 
<math>f(x)</math> must have four roots, three of which are roots of <math>g(x)</math>. Using the fact that every polynomial has a unique factorization into its roots, and since the leading coefficient of <math>f(x)</math> and <math>g(x)</math> are the same, we know that
 
<math>f(x)</math> must have four roots, three of which are roots of <math>g(x)</math>. Using the fact that every polynomial has a unique factorization into its roots, and since the leading coefficient of <math>f(x)</math> and <math>g(x)</math> are the same, we know that
  
Line 22: Line 22:
 
-10r&=c.\\
 
-10r&=c.\\
 
\end{align*}</cmath>
 
\end{align*}</cmath>
 +
 +
(Solution 1.1 picks up here.)
  
 
Let's solve for <math>a,b,c,</math> and <math>r</math>. Since <math>10-r=100</math>, <math>r=-90</math>, so <math>c=(-10)(-90)=900</math>. Since <math>a-r=1</math>, <math>a=-89</math>, and <math>b=1-ar=-8009</math>. Thus, we know that
 
Let's solve for <math>a,b,c,</math> and <math>r</math>. Since <math>10-r=100</math>, <math>r=-90</math>, so <math>c=(-10)(-90)=900</math>. Since <math>a-r=1</math>, <math>a=-89</math>, and <math>b=1-ar=-8009</math>. Thus, we know that
Line 32: Line 34:
 
f(1)&=1^4+1^3-8009(1)^2+100(1)+900\\
 
f(1)&=1^4+1^3-8009(1)^2+100(1)+900\\
 
&=1+1-8009+100+900\\
 
&=1+1-8009+100+900\\
&=\boxed{\bold{(C)\text{ }}\text{-}7007}.\\
+
&=\boxed{\bold{(C)}\, -7007}.\\
 
\end{align*}</cmath>
 
\end{align*}</cmath>
 +
 +
==Solution 1.1==
 +
A faster ending to Solution 1 is as follows.
 +
We shall solve for only <math>a</math> and <math>r</math>. Since <math>10-r=100</math>, <math>r=-90</math>, and since <math>a-r=1</math>, <math>a=-89</math>. Then,
 +
<cmath>\begin{align*}
 +
f(1)&=(1-r)(1^3+a\cdot1^2+1+10)\\
 +
&=(91)(-77)\\
 +
&=\boxed{\bold{(C)}\, -7007}.\\
 +
\end{align*}</cmath>
 +
 +
==Solution 1.2 (Fast)==
 +
Let the term <math>q(x)</math> be the linear term that we are solving for in the equation <math>f(x)=g(x)\cdot q(x)</math>. Now, we know that <math>q(x)=mx-r</math> must have <math>m=1</math>, because only <math>x \cdot x^3=x^4</math>. In addition, we know that, by distributing, <math>10x-rx=100x</math>. Therefore, <math>r=-90</math>, and all the other variables are quickly solved for.
  
 
==Solution 2==
 
==Solution 2==
Line 55: Line 69:
 
<cmath>1+\frac{ac}{10} = b = 1+\frac{-89 \cdot 900}{10} = b = -8009</cmath>.
 
<cmath>1+\frac{ac}{10} = b = 1+\frac{-89 \cdot 900}{10} = b = -8009</cmath>.
  
We know that <math>f(1)</math> is the sum of its coefficients, hence <math>1+1+b+100+c</math>. We substitute the values we obtained for <math>b</math> and <math>c</math> into this expression to get <math>f(1) = 1 + 1 + (-8009) + 100 + 900 = \boxed{-7007}</math> or <math>\boxed{\text{C}}</math>.
+
We know that <math>f(1)</math> is the sum of its coefficients, hence <math>1+1+b+100+c</math>. We substitute the values we obtained for <math>b</math> and <math>c</math> into this expression to get <math>f(1) = 1 + 1 + (-8009) + 100 + 900 = \boxed{\textbf{(C)}\,-7007}</math>.
 +
 
 +
==Solution 3==
 +
 
 +
Let <math>r_1,r_2,</math> and <math>r_3</math> be the roots of <math>g(x)</math>. Let <math>r_4</math> be the additional root of <math>f(x)</math>. Then from Vieta's formulas on the quadratic term of <math>g(x)</math> and the cubic term of <math>f(x)</math>, we obtain the following:
 +
 
 +
<cmath>\begin{align*}
 +
r_1+r_2+r_3&=-a \\
 +
r_1+r_2+r_3+r_4&=-1
 +
\end{align*}</cmath>
 +
 
 +
Thus <math>r_4=a-1</math>.
 +
 
 +
Now applying Vieta's formulas on the constant term of <math>g(x)</math>, the linear term of <math>g(x)</math>, and the linear term of <math>f(x)</math>, we obtain:
 +
 
 +
<cmath>\begin{align*}
 +
r_1r_2r_3  & = -10\\
 +
r_1r_2+r_2r_3+r_3r_1 &= 1\\
 +
r_1r_2r_3+r_2r_3r_4+r_3r_4r_1+r_4r_1r_2  & = -100\\
 +
\end{align*}</cmath>
 +
 
 +
Substituting for <math>r_1r_2r_3</math> in the bottom equation and factoring the remainder of the expression, we obtain:
 +
 
 +
<cmath>-10+(r_1r_2+r_2r_3+r_3r_1)r_4=-10+r_4=-100</cmath>
 +
 
 +
It follows that <math>r_4=-90</math>. But <math>r_4=a-1</math> so <math>a=-89</math>
 +
 
 +
Now we can factor <math>f(x)</math> in terms of <math>g(x)</math> as
 +
 
 +
<cmath>f(x)=(x-r_4)g(x)=(x+90)g(x)</cmath>
 +
 
 +
Then <math>f(1)=91g(1)</math> and
 +
 
 +
<cmath>g(1)=1^3-89\cdot 1^2+1+10=-77</cmath>
 +
 
 +
Hence <math>f(1)=91\cdot(-77)=\boxed{\textbf{(C)}\,-7007}</math>.
 +
 
 +
==Solution 4 (Risky)==
 +
Let the roots of <math>g(x)</math> be <math>r_1</math>, <math>r_2</math>, and <math>r_3</math>. Let the roots of <math>f(x)</math> be <math>r_1</math>, <math>r_2</math>, <math>r_3</math>, and <math>r_4</math>. From Vieta's, we have:
 +
<cmath>\begin{align*}
 +
r_1+r_2+r_3=-a \\
 +
r_1+r_2+r_3+r_4=-1 \\
 +
r_4=a-1
 +
\end{align*}</cmath>
 +
The fourth root is <math>a-1</math>. Since <math>r_1</math>, <math>r_2</math>, and <math>r_3</math> are common roots, we have:
 +
<cmath>\begin{align*}
 +
f(x)=g(x)(x-(a-1)) \\
 +
f(1)=g(1)(1-(a-1)) \\
 +
f(1)=(a+12)(2-a) \\
 +
f(1)=-(a+12)(a-2) \\
 +
\end{align*}</cmath>
 +
Let <math>a-2=k</math>:
 +
<cmath>\begin{align*}
 +
f(1)=-k(k+14)
 +
\end{align*}</cmath>
 +
Note that <math>-7007=-1001\cdot(7)=-(7\cdot(11)\cdot(13))\cdot(7)=-91\cdot(77)</math>
 +
This gives us a pretty good guess of <math>\boxed{\textbf{(C)}\, -7007}</math>.
 +
 
 +
==Solution 5==
 +
First off, let's get rid of the <math>x^4</math> term by finding <math>h(x)=f(x)-xg(x)</math>. This polynomial consists of the difference of two polynomials with <math>3</math> common factors, so it must also have these factors. The polynomial is <math>h(x)=(1-a)x^3 + (b-1)x^2 + 90x + c</math>, and must be equal to <math>(1-a)g(x)</math>. Equating the coefficients, we get <math>3</math> equations. We will tackle the situation one equation at a time, starting the <math>x</math> terms. Looking at the coefficients, we get <math>\dfrac{90}{1-a} = 1</math>. <cmath>\therefore 90=1-a.</cmath> The solution to the previous is obviously <math>a=-89</math>. We can now find <math>b</math> and <math>c</math>. <math>\dfrac{b-1}{1-a} = a</math>, <cmath>\therefore b-1=a(1-a)=-89*90=-8010</cmath> and <math>b=-8009</math>. Finally <math>\dfrac{c}{1-a} = 10</math>, <cmath>\therefore c=10(1-a)=10*90=900</cmath> Solving the original problem, <math>f(1)=1 + 1 + b + 100 + c = 102+b+c=102+900-8009=\boxed{\textbf{(C)}\, -7007}</math>.
 +
 
 +
==Solution 6==
 +
Simple polynomial division is a feasible method. Even though we have variables, we can equate terms at the end of the division so that we can cancel terms. Doing the division of <math>\frac{f(x)}{g(x)}</math> eventually brings us the final step <math>(1-a)x^3 + (b-1)x^2 + 90x + c</math> minus <math>(1-a)x^3 - (a-a^2)x^2 + (1-a)x + 10(1-a)</math> after we multiply <math>f(x)</math> by <math>(1-a)</math>. Now we equate coefficients of same-degree <math>x</math> terms. This gives us <math> 10(1-a) = c, b-1 = a - a^2, 1-a = 90 \Rightarrow a = -89, c = 900, b = -8009</math>. We are interested in finding <math>f(1)</math>, which equals <math>1^4 + 1^3 -8009\cdot1^2 + 100\cdot1 + 900 = \boxed{\textbf{(C)}\,-7007}</math>. ~skyscraper
 +
 
 +
==General Note==
 +
 
 +
Note that <math>f(1)</math> for any polynomial is simply the sum of the coefficients of the polynomial.
 +
 
 +
==Video Solution==
 +
https://youtu.be/wXvRNC-48Lk
 +
 
 +
https://www.youtube.com/watch?v=MBIiz0mroqk  (By Richard Rusczyk)
 +
 
 +
https://youtu.be/3dfbWzOfJAI?t=4412
 +
 
 +
~ pi_is_3.14
  
 
==See Also==
 
==See Also==
{{AMC10 box|year=2017|ab=A|num-b=8|num-a=10}}
+
{{AMC10 box|year=2017|ab=A|num-b=23|num-a=25}}
 +
{{AMC12 box|year=2017|ab=A|num-b=22|num-a=24}}
 
{{MAA Notice}}
 
{{MAA Notice}}
 +
 +
[[Category:Intermediate Algebra Problems]]

Revision as of 03:11, 28 January 2021

Problem

For certain real numbers $a$, $b$, and $c$, the polynomial \[g(x) = x^3 + ax^2 + x + 10\]has three distinct roots, and each root of $g(x)$ is also a root of the polynomial \[f(x) = x^4 + x^3 + bx^2 + 100x + c.\]What is $f(1)$?

$\textbf{(A)}\ -9009 \qquad\textbf{(B)}\ -8008 \qquad\textbf{(C)}\ -7007 \qquad\textbf{(D)}\ -6006 \qquad\textbf{(E)}\ -5005$

Solution 1

$f(x)$ must have four roots, three of which are roots of $g(x)$. Using the fact that every polynomial has a unique factorization into its roots, and since the leading coefficient of $f(x)$ and $g(x)$ are the same, we know that

\[f(x)=g(x)(x-r)\]

where $r\in\mathbb{C}$ is the fourth root of $f(x)$. Substituting $g(x)$ and expanding, we find that

\begin{align*}f(x)&=(x^3+ax^2+x+10)(x-r)\\ &=x^4+(a-r)x^3+(1-ar)x^2+(10-r)x-10r.\end{align*}

Comparing coefficients with $f(x)$, we see that

\begin{align*} a-r&=1\\ 1-ar&=b\\ 10-r&=100\\ -10r&=c.\\ \end{align*}

(Solution 1.1 picks up here.)

Let's solve for $a,b,c,$ and $r$. Since $10-r=100$, $r=-90$, so $c=(-10)(-90)=900$. Since $a-r=1$, $a=-89$, and $b=1-ar=-8009$. Thus, we know that

\[f(x)=x^4+x^3-8009x^2+100x+900.\]

Taking $f(1)$, we find that

\begin{align*} f(1)&=1^4+1^3-8009(1)^2+100(1)+900\\ &=1+1-8009+100+900\\ &=\boxed{\bold{(C)}\, -7007}.\\ \end{align*}

Solution 1.1

A faster ending to Solution 1 is as follows. We shall solve for only $a$ and $r$. Since $10-r=100$, $r=-90$, and since $a-r=1$, $a=-89$. Then, \begin{align*} f(1)&=(1-r)(1^3+a\cdot1^2+1+10)\\ &=(91)(-77)\\ &=\boxed{\bold{(C)}\, -7007}.\\ \end{align*}

Solution 1.2 (Fast)

Let the term $q(x)$ be the linear term that we are solving for in the equation $f(x)=g(x)\cdot q(x)$. Now, we know that $q(x)=mx-r$ must have $m=1$, because only $x \cdot x^3=x^4$. In addition, we know that, by distributing, $10x-rx=100x$. Therefore, $r=-90$, and all the other variables are quickly solved for.

Solution 2

We notice that the constant term of $f(x)=c$ and the constant term in $g(x)=10$. Because $f(x)$ can be factored as $g(x) \cdot (x- r)$ (where $r$ is the unshared root of $f(x)$, we see that using the constant term, $-10 \cdot r = c$ and therefore $r = -\frac{c}{10}$. Now we once again write $f(x)$ out in factored form:

\[f(x) = g(x)\cdot (x-r) = (x^3+ax^2+x+10)(x+\frac{c}{10})\].

We can expand the expression on the right-hand side to get:

\[f(x) = x^4+(a+\frac{c}{10})x^3+(1+\frac{ac}{10})x^2+(10+\frac{c}{10})x+c\]

Now we have $f(x) = x^4+(a+\frac{c}{10})x^3+(1+\frac{ac}{10})x^2+(10+\frac{c}{10})x+c=x^4+x^3+bx^2+100x+c$.

Simply looking at the coefficients for each corresponding term (knowing that they must be equal), we have the equations: \[10+\frac{c}{10}=100 \Rightarrow c=900\] \[a+\frac{c}{10} = 1, c=900 \Rightarrow a + 90 =1 \Rightarrow a= -89\]

and finally,

\[1+\frac{ac}{10} = b = 1+\frac{-89 \cdot 900}{10} = b = -8009\].

We know that $f(1)$ is the sum of its coefficients, hence $1+1+b+100+c$. We substitute the values we obtained for $b$ and $c$ into this expression to get $f(1) = 1 + 1 + (-8009) + 100 + 900 = \boxed{\textbf{(C)}\,-7007}$.

Solution 3

Let $r_1,r_2,$ and $r_3$ be the roots of $g(x)$. Let $r_4$ be the additional root of $f(x)$. Then from Vieta's formulas on the quadratic term of $g(x)$ and the cubic term of $f(x)$, we obtain the following:

\begin{align*} r_1+r_2+r_3&=-a \\  r_1+r_2+r_3+r_4&=-1 \end{align*}

Thus $r_4=a-1$.

Now applying Vieta's formulas on the constant term of $g(x)$, the linear term of $g(x)$, and the linear term of $f(x)$, we obtain:

\begin{align*} r_1r_2r_3  & = -10\\ r_1r_2+r_2r_3+r_3r_1 &= 1\\  r_1r_2r_3+r_2r_3r_4+r_3r_4r_1+r_4r_1r_2  & = -100\\ \end{align*}

Substituting for $r_1r_2r_3$ in the bottom equation and factoring the remainder of the expression, we obtain:

\[-10+(r_1r_2+r_2r_3+r_3r_1)r_4=-10+r_4=-100\]

It follows that $r_4=-90$. But $r_4=a-1$ so $a=-89$

Now we can factor $f(x)$ in terms of $g(x)$ as

\[f(x)=(x-r_4)g(x)=(x+90)g(x)\]

Then $f(1)=91g(1)$ and

\[g(1)=1^3-89\cdot 1^2+1+10=-77\]

Hence $f(1)=91\cdot(-77)=\boxed{\textbf{(C)}\,-7007}$.

Solution 4 (Risky)

Let the roots of $g(x)$ be $r_1$, $r_2$, and $r_3$. Let the roots of $f(x)$ be $r_1$, $r_2$, $r_3$, and $r_4$. From Vieta's, we have: \begin{align*} r_1+r_2+r_3=-a \\ r_1+r_2+r_3+r_4=-1 \\ r_4=a-1 \end{align*} The fourth root is $a-1$. Since $r_1$, $r_2$, and $r_3$ are common roots, we have: \begin{align*} f(x)=g(x)(x-(a-1)) \\ f(1)=g(1)(1-(a-1)) \\ f(1)=(a+12)(2-a) \\ f(1)=-(a+12)(a-2) \\ \end{align*} Let $a-2=k$: \begin{align*} f(1)=-k(k+14) \end{align*} Note that $-7007=-1001\cdot(7)=-(7\cdot(11)\cdot(13))\cdot(7)=-91\cdot(77)$ This gives us a pretty good guess of $\boxed{\textbf{(C)}\, -7007}$.

Solution 5

First off, let's get rid of the $x^4$ term by finding $h(x)=f(x)-xg(x)$. This polynomial consists of the difference of two polynomials with $3$ common factors, so it must also have these factors. The polynomial is $h(x)=(1-a)x^3 + (b-1)x^2 + 90x + c$, and must be equal to $(1-a)g(x)$. Equating the coefficients, we get $3$ equations. We will tackle the situation one equation at a time, starting the $x$ terms. Looking at the coefficients, we get $\dfrac{90}{1-a} = 1$. \[\therefore 90=1-a.\] The solution to the previous is obviously $a=-89$. We can now find $b$ and $c$. $\dfrac{b-1}{1-a} = a$, \[\therefore b-1=a(1-a)=-89*90=-8010\] and $b=-8009$. Finally $\dfrac{c}{1-a} = 10$, \[\therefore c=10(1-a)=10*90=900\] Solving the original problem, $f(1)=1 + 1 + b + 100 + c = 102+b+c=102+900-8009=\boxed{\textbf{(C)}\, -7007}$.

Solution 6

Simple polynomial division is a feasible method. Even though we have variables, we can equate terms at the end of the division so that we can cancel terms. Doing the division of $\frac{f(x)}{g(x)}$ eventually brings us the final step $(1-a)x^3 + (b-1)x^2 + 90x + c$ minus $(1-a)x^3 - (a-a^2)x^2 + (1-a)x + 10(1-a)$ after we multiply $f(x)$ by $(1-a)$. Now we equate coefficients of same-degree $x$ terms. This gives us $10(1-a) = c, b-1 = a - a^2, 1-a = 90 \Rightarrow a = -89, c = 900, b = -8009$. We are interested in finding $f(1)$, which equals $1^4 + 1^3 -8009\cdot1^2 + 100\cdot1 + 900 = \boxed{\textbf{(C)}\,-7007}$. ~skyscraper

General Note

Note that $f(1)$ for any polynomial is simply the sum of the coefficients of the polynomial.

Video Solution

https://youtu.be/wXvRNC-48Lk

https://www.youtube.com/watch?v=MBIiz0mroqk (By Richard Rusczyk)

https://youtu.be/3dfbWzOfJAI?t=4412

~ pi_is_3.14

See Also

2017 AMC 10A (ProblemsAnswer KeyResources)
Preceded by
Problem 23
Followed by
Problem 25
1 2 3 4 5 6 7 8 9 10 11 12 13 14 15 16 17 18 19 20 21 22 23 24 25
All AMC 10 Problems and Solutions
2017 AMC 12A (ProblemsAnswer KeyResources)
Preceded by
Problem 22
Followed by
Problem 24
1 2 3 4 5 6 7 8 9 10 11 12 13 14 15 16 17 18 19 20 21 22 23 24 25
All AMC 12 Problems and Solutions

The problems on this page are copyrighted by the Mathematical Association of America's American Mathematics Competitions. AMC logo.png